Third Party

This topic has expert replies
Legendary Member
Posts: 2326
Joined: Mon Jul 28, 2008 3:54 am
Thanked: 173 times
Followed by:2 members
GMAT Score:710

Third Party

by gmatmachoman » Tue Sep 22, 2009 10:01 am
In a political system with only two major parties, the entrance of a third-party candidate into an election race damages the chances of only one of the two major candidates. The third-party candidate always attracts some of the voters who might otherwise have voted for one of the two major candidates, but not voters who support the other candidate. Since a third-party candidacy affects the two major candidates unequally, for reasons neither of them has any control over, the practice is unfair and should not be allowed.
If the factual information in the passage above is true, which of the following can be most reliably inferred from it?

(A) If the political platform of the third party is a compromise position between that of the two major parties, the third party will draw its voters equally from the two major parties.
(B) If, before the emergence of a third party, voters were divided equally between the two major parties, neither of the major parties is likely to capture much more than one-half of the vote.
(C) A third-party candidate will not capture the votes of new voters who have never voted for candidates of either of the two major parties.
(D) The political stance of a third party will be more radical than that of either of the two major parties.
(E) The founders of a third party are likely to be a coalition consisting of former leaders of the two major parties.

OA [spoiler]after some discussions!![/spoiler]

Master | Next Rank: 500 Posts
Posts: 160
Joined: Sun May 03, 2009 1:17 am
Location: Rourkela/Hyderabad
Thanked: 4 times
Followed by:1 members

by sanp_l » Tue Sep 22, 2009 10:24 am
I go with option B.
Sandy

Master | Next Rank: 500 Posts
Posts: 256
Joined: Mon Aug 10, 2009 6:31 pm
Thanked: 3 times

by gmatv09 » Tue Sep 22, 2009 10:53 am
IMO B

two parties have equal voters. introduction of third party, will cause some of the voters from one of the political party will vote for the third party....

Legendary Member
Posts: 2326
Joined: Mon Jul 28, 2008 3:54 am
Thanked: 173 times
Followed by:2 members
GMAT Score:710

by gmatmachoman » Fri Sep 25, 2009 6:19 am
Its not B


sorry!!

Anyone still there?

Plz use spoilers & do drop some useful explanation even if other user has dropped one..

We are here to read your comments nad explanations..
U COULD BUILD "UR READERS" LIKE OUR MODERATORS LIKE DANAJ,CR AND OUR GMAT PREP TUTORS WHOS GYAN IS SO ENRICHING AND MAKES ME TO SIT AND READ & UNDERSTAND...

Newbie | Next Rank: 10 Posts
Posts: 9
Joined: Fri Nov 21, 2008 3:29 am
Thanked: 1 times

by gmatnvarun » Sat Sep 26, 2009 3:38 am
imo A...the statemnt in question says that right now,the third party is expected to have votes only from on the the 2 major parties,the party towards which the voters of third party are bent....but had the third party had equal number of voters for each of the 2 major parties,the case would have been what is stated in A

Legendary Member
Posts: 527
Joined: Mon Jun 02, 2008 9:14 am
Location: Atlanta
Thanked: 17 times

by pandeyvineet24 » Sat Sep 26, 2009 12:24 pm
IMO B and C are 2 contenders.
I chose C over B. The issue with B is that it starts with equal votes for 2 existing parties, but then in the second part of the sentence it says not much more than half of the votes. It should have said exactly half the number of votes.

Legendary Member
Posts: 1404
Joined: Tue May 20, 2008 6:55 pm
Thanked: 18 times
Followed by:2 members

by tanviet » Wed Sep 30, 2009 12:52 am
inference, according to "critical REasoning bible" , is infered from 2 pieces of information in the arguement or is old information which is paraphrased.

in this case, inference is old information paraphrased. C must be correct.

Newbie | Next Rank: 10 Posts
Posts: 5
Joined: Thu Jul 02, 2009 10:36 am

by bluecollarhero » Wed Sep 30, 2009 12:14 pm
duongthang wrote:inference, according to "critical REasoning bible" , is infered from 2 pieces of information in the arguement or is old information which is paraphrased.

in this case, inference is old information paraphrased. C must be correct.
I am not sure what you mean by old information, but I think C is out of scope. The paragraph states that the 3rd party candidats always takes some votes from one of the major candidates, but it is mute on new voters who have not voted before. Plus the use of 'not' is sort of extreme.

My vote is for B. I like A, but it requires a huge leap of faith, which inference questions are not meant to do.

Master | Next Rank: 500 Posts
Posts: 239
Joined: Wed Feb 11, 2009 2:50 am

by delhiboy1979 » Thu Oct 01, 2009 12:33 am
Not sure why B is not the answer here. Afterall, the passage does mention that the third party can only attract voters who are not sure of a party to vote for. And that is what B confirms. If the voters were sure of which party they need to vote for, there would be nothing for the third party to attract.

Junior | Next Rank: 30 Posts
Posts: 24
Joined: Wed Sep 30, 2009 3:41 am
Thanked: 2 times

by hypermeganet » Thu Oct 01, 2009 9:41 am
A. If the third party has a position that is 50% party A and 50% party B it'll attract an equal number from each side per the stimulus.

Senior | Next Rank: 100 Posts
Posts: 73
Joined: Sat Oct 03, 2009 10:37 pm

my opinion!

by kiennguyen » Mon Oct 05, 2009 5:02 am
A-no evidence
B-Untrue, if two parties have the same vote, no party will win the election.
D-no evidence
E-no evidence

C should be the answer b/c "The third-party candidate always attracts some of the voters who might otherwise have voted for one of the two major candidates, but not voters who support the other candidate."
Actually, C didn't make much sense to me but little better than the others.
If anyone knows the answer and the better explanation, please help us out!

Master | Next Rank: 500 Posts
Posts: 295
Joined: Tue Jul 15, 2008 10:07 am
Thanked: 4 times
GMAT Score:690

by vaibhav.iit2002 » Mon Oct 05, 2009 6:42 am
IMO B[this isn't best choice but seems least flawed

OA plz

Legendary Member
Posts: 869
Joined: Wed Aug 26, 2009 3:49 pm
Location: California
Thanked: 13 times
Followed by:3 members

Re: Third Party

by heshamelaziry » Fri Oct 09, 2009 9:22 pm
gmatmachoman wrote:In a political system with only two major parties, the entrance of a third-party candidate into an election race damages the chances of only one of the two major candidates. The third-party candidate always attracts some of the voters who might otherwise have voted for one of the two major candidates, but not voters who support the other candidate. Since a third-party candidacy affects the two major candidates unequally, for reasons neither of them has any control over, the practice is unfair and should not be allowed.
If the factual information in the passage above is true, which of the following can be most reliably inferred from it?

(A) If the political platform of the third party is a compromise position between that of the two major parties, the third party will draw its voters equally from the two major parties.
(B) If, before the emergence of a third party, voters were divided equally between the two major parties, neither of the major parties is likely to capture much more than one-half of the vote.
(C) A third-party candidate will not capture the votes of new voters who have never voted for candidates of either of the two major parties.
(D) The political stance of a third party will be more radical than that of either of the two major parties.
(E) The founders of a third party are likely to be a coalition consisting of former leaders of the two major parties.

OA [spoiler]after some discussions!![/spoiler]
OA Please

Legendary Member
Posts: 2326
Joined: Mon Jul 28, 2008 3:54 am
Thanked: 173 times
Followed by:2 members
GMAT Score:710

by gmatmachoman » Sat Oct 10, 2009 9:39 pm

Legendary Member
Posts: 727
Joined: Sun Jun 08, 2008 9:32 pm
Thanked: 8 times
Followed by:1 members

by umaa » Sat Oct 10, 2009 9:44 pm
Am I looking something wrong? I just found the OA as B through Google.

Let me know what is the source of the question?
What we think, we become